Norm of a symmetric matrix equals spectral radius












24












$begingroup$


How do I prove that the norm of a matrix equals the absolutely largest eigenvalue of the matrix? This is the precise question:




Let $A$ be a symmetric $n times n$ matrix. Consider $A$ as an operator in $mathbb{R}^n$ given by $x mapsto Ax$. Prove that $|A| = max_j |lambda_j|$, where $lambda_j$ are the eigenvalues of $A$.




I've read the relevant sections in my literature over and over but can't find any clue on how to begin. A solution is suggested here but the notion of diagonal operator is not in my literature so it doesn't tell me very much. So, any other hints on how to solve the question? Thanks.










share|cite|improve this question











$endgroup$

















    24












    $begingroup$


    How do I prove that the norm of a matrix equals the absolutely largest eigenvalue of the matrix? This is the precise question:




    Let $A$ be a symmetric $n times n$ matrix. Consider $A$ as an operator in $mathbb{R}^n$ given by $x mapsto Ax$. Prove that $|A| = max_j |lambda_j|$, where $lambda_j$ are the eigenvalues of $A$.




    I've read the relevant sections in my literature over and over but can't find any clue on how to begin. A solution is suggested here but the notion of diagonal operator is not in my literature so it doesn't tell me very much. So, any other hints on how to solve the question? Thanks.










    share|cite|improve this question











    $endgroup$















      24












      24








      24


      24



      $begingroup$


      How do I prove that the norm of a matrix equals the absolutely largest eigenvalue of the matrix? This is the precise question:




      Let $A$ be a symmetric $n times n$ matrix. Consider $A$ as an operator in $mathbb{R}^n$ given by $x mapsto Ax$. Prove that $|A| = max_j |lambda_j|$, where $lambda_j$ are the eigenvalues of $A$.




      I've read the relevant sections in my literature over and over but can't find any clue on how to begin. A solution is suggested here but the notion of diagonal operator is not in my literature so it doesn't tell me very much. So, any other hints on how to solve the question? Thanks.










      share|cite|improve this question











      $endgroup$




      How do I prove that the norm of a matrix equals the absolutely largest eigenvalue of the matrix? This is the precise question:




      Let $A$ be a symmetric $n times n$ matrix. Consider $A$ as an operator in $mathbb{R}^n$ given by $x mapsto Ax$. Prove that $|A| = max_j |lambda_j|$, where $lambda_j$ are the eigenvalues of $A$.




      I've read the relevant sections in my literature over and over but can't find any clue on how to begin. A solution is suggested here but the notion of diagonal operator is not in my literature so it doesn't tell me very much. So, any other hints on how to solve the question? Thanks.







      matrices functional-analysis operator-theory symmetric-matrices spectral-radius






      share|cite|improve this question















      share|cite|improve this question













      share|cite|improve this question




      share|cite|improve this question








      edited Jul 8 '18 at 22:57









      Rodrigo de Azevedo

      13.1k41960




      13.1k41960










      asked Dec 11 '13 at 21:57









      Erik VesterlundErik Vesterlund

      6751816




      6751816






















          2 Answers
          2






          active

          oldest

          votes


















          21












          $begingroup$

          The norm of a matrix is defined as
          begin{equation}
          |A| = sup_{|u| = 1} |Au|
          end{equation}

          Taking the singular value decomposition of the matrix $A$, we have
          begin{equation}
          A = VD W^T
          end{equation}

          where $V$ and $W$ are orthonormal and $D$ is a diagonal matrix. Since $V$ and $W$ are orthonormal, we have $|V| = 1$ and $|W| = 1$. Then $|Av| = |D v|$ for any vector $v$. Then we can maximize the norm of $Av$ by maximizing the norm of $Dv$.



          By the definition of singular value decomposition, $D$ will have the singular values of $A$ on its main diagonal and will have zeros everywhere else. Let $lambda_1, ldots, lambda_n$ denote these diagonal entries so that



          begin{equation}
          D = left(begin{array}{cccc}
          lambda_1 & 0 & ldots & 0 \
          0 & lambda_2 & ldots & 0 \
          vdots & & ddots & vdots \
          0 & 0 & ldots & lambda_n
          end{array}right)
          end{equation}



          Taking some $v = (v_1, v_2, ldots, v_n)^T$, the product $Dv$ takes the form
          begin{equation}
          Dv = left(begin{array}{c}
          lambda_1v_1 \
          vdots \
          lambda_nv_n
          end{array}right)
          end{equation}

          Maximizing the norm of this is the same as maximizing the norm squared. Then we are trying to maximize the sum
          begin{equation}
          S = sum_{i=1}^{n} lambda_i^2v_i^2
          end{equation}

          under the constraint that $v$ is a unit vector (i.e., $sum_i v_i^2 = 1$). The maximum is attained by finding the largest $lambda_i^2$ and setting its corresponding $v_i$ to $1$ and then setting each other $v_j$ to $0$. Then the maximum of $S$ (which is the norm squared) is the square of the absolutely largest eigenvalue of $A$. Taking the square root, we get the absolutely largest eigenvalue of $A$.






          share|cite|improve this answer











          $endgroup$













          • $begingroup$
            "Since $Dv$ has only nonzero entries..." I don't understand much in this section. Why is the maximum of $||Dv||$ attained when we select $e_n$, what does "pick off" refer to, and what's SVD?
            $endgroup$
            – Erik Vesterlund
            Dec 11 '13 at 22:38










          • $begingroup$
            I've added some details (and removed a few confusing things) to make this a little clearer.
            $endgroup$
            – yoknapatawpha
            Dec 12 '13 at 0:10










          • $begingroup$
            This is overkill. See @user42070's more elementary approach.
            $endgroup$
            – Ted Shifrin
            Dec 12 '13 at 0:38






          • 1




            $begingroup$
            @TedShifrin is it not up to me who asked the question to determine what is overkill? I asked for more details, the answer is not overkill, it's precisely what I needed.
            $endgroup$
            – Erik Vesterlund
            Dec 12 '13 at 13:50






          • 1




            $begingroup$
            @ErikVesterlund If you consider $S$ just as a weighted sum (putting aside for now that these are eigenvalues of $A$ we're dealing with), then intuitively it makes sense that to maximize the weighted sum, we'd assign all of the weight to the largest element in the sum. Considering the case of a $2 times 2$ matrix, we'd just have $lambda_1$ and $lambda_2$ so that $S = lambda_1^2v_1^2 + lambda_2^2v_2^2$. Here, if $lambda_1 > lambda_2$, we'd maximize $S$ by setting $v_1 = 1$ and $v_2 = 0$ (and vice-versa if $lambda_2 > lambda_1$). The case of higher dimensions is similar. Does this help?
            $endgroup$
            – yoknapatawpha
            Dec 12 '13 at 20:33





















          15












          $begingroup$

          But the key point is exactly that your matrix is diagonalizable more specially that you can find an orthonormal basis of eigenvector $e_i$ for which you have $A e_i=lambda_i e_i$.



          Then your write for $x=sum x_ie_i$ and you have $Ax=sum x_ilambda_i e_i$ so that $|Ax|^2=sumlambda_i^2x_i^2$ now by definition of the norm of matrix it gives you $$frac{|Ax|}{|x|}leq|lambda_{i_0}|$$ therefore $|A|leq|lambda_{i_0}|$ where $lambda_{i_0}$ is the greatest eigenvalue . Finally the identity $|Ae_{i_0}|=|lambda_{i_0}e_i|$ gives you the inequality $|A|geq|lambda_{i_0}|$.






          share|cite|improve this answer











          $endgroup$













          • $begingroup$
            How do you get that $||Ax||^2 = sum lambda_i^2 x_i^2$? At least in my book the matrix norm is not defined, not previous to the exercise anyway so, what is it?
            $endgroup$
            – Erik Vesterlund
            Dec 11 '13 at 22:36






          • 1




            $begingroup$
            just because $(e_i)$ is an orthonormal basis, the whole question relies on the euclidean structure : the norm in $mathbb{R}^n$ is the one defined by $|x|^2:=<x,x>=sum x_i^2$. The fact that $e_i$ is orthonormal reads $<e_i,e_j>=delta_{ij}$ which means $1$ if $i=j$ and $0$ otherwise. Now compute $|x|^2=<x,x>=sum x_ix_j<e_i,e_j>=sum x_i^2$
            $endgroup$
            – user42070
            Dec 11 '13 at 22:44












          • $begingroup$
            Is it correct to draw the conclusion, that when we can find an eigenbasis, then the largest singular value always equals the largest eigenvalue?
            $endgroup$
            – Thomas Ahle
            Feb 14 '17 at 6:24










          • $begingroup$
            So $x_i$ is not the elements of $x$ but its decomposition in the orthonormal basis $(e_i)$?
            $endgroup$
            – Ella Shar
            Jan 27 at 8:55














          Your Answer





          StackExchange.ifUsing("editor", function () {
          return StackExchange.using("mathjaxEditing", function () {
          StackExchange.MarkdownEditor.creationCallbacks.add(function (editor, postfix) {
          StackExchange.mathjaxEditing.prepareWmdForMathJax(editor, postfix, [["$", "$"], ["\\(","\\)"]]);
          });
          });
          }, "mathjax-editing");

          StackExchange.ready(function() {
          var channelOptions = {
          tags: "".split(" "),
          id: "69"
          };
          initTagRenderer("".split(" "), "".split(" "), channelOptions);

          StackExchange.using("externalEditor", function() {
          // Have to fire editor after snippets, if snippets enabled
          if (StackExchange.settings.snippets.snippetsEnabled) {
          StackExchange.using("snippets", function() {
          createEditor();
          });
          }
          else {
          createEditor();
          }
          });

          function createEditor() {
          StackExchange.prepareEditor({
          heartbeatType: 'answer',
          autoActivateHeartbeat: false,
          convertImagesToLinks: true,
          noModals: true,
          showLowRepImageUploadWarning: true,
          reputationToPostImages: 10,
          bindNavPrevention: true,
          postfix: "",
          imageUploader: {
          brandingHtml: "Powered by u003ca class="icon-imgur-white" href="https://imgur.com/"u003eu003c/au003e",
          contentPolicyHtml: "User contributions licensed under u003ca href="https://creativecommons.org/licenses/by-sa/3.0/"u003ecc by-sa 3.0 with attribution requiredu003c/au003e u003ca href="https://stackoverflow.com/legal/content-policy"u003e(content policy)u003c/au003e",
          allowUrls: true
          },
          noCode: true, onDemand: true,
          discardSelector: ".discard-answer"
          ,immediatelyShowMarkdownHelp:true
          });


          }
          });














          draft saved

          draft discarded


















          StackExchange.ready(
          function () {
          StackExchange.openid.initPostLogin('.new-post-login', 'https%3a%2f%2fmath.stackexchange.com%2fquestions%2f603375%2fnorm-of-a-symmetric-matrix-equals-spectral-radius%23new-answer', 'question_page');
          }
          );

          Post as a guest















          Required, but never shown

























          2 Answers
          2






          active

          oldest

          votes








          2 Answers
          2






          active

          oldest

          votes









          active

          oldest

          votes






          active

          oldest

          votes









          21












          $begingroup$

          The norm of a matrix is defined as
          begin{equation}
          |A| = sup_{|u| = 1} |Au|
          end{equation}

          Taking the singular value decomposition of the matrix $A$, we have
          begin{equation}
          A = VD W^T
          end{equation}

          where $V$ and $W$ are orthonormal and $D$ is a diagonal matrix. Since $V$ and $W$ are orthonormal, we have $|V| = 1$ and $|W| = 1$. Then $|Av| = |D v|$ for any vector $v$. Then we can maximize the norm of $Av$ by maximizing the norm of $Dv$.



          By the definition of singular value decomposition, $D$ will have the singular values of $A$ on its main diagonal and will have zeros everywhere else. Let $lambda_1, ldots, lambda_n$ denote these diagonal entries so that



          begin{equation}
          D = left(begin{array}{cccc}
          lambda_1 & 0 & ldots & 0 \
          0 & lambda_2 & ldots & 0 \
          vdots & & ddots & vdots \
          0 & 0 & ldots & lambda_n
          end{array}right)
          end{equation}



          Taking some $v = (v_1, v_2, ldots, v_n)^T$, the product $Dv$ takes the form
          begin{equation}
          Dv = left(begin{array}{c}
          lambda_1v_1 \
          vdots \
          lambda_nv_n
          end{array}right)
          end{equation}

          Maximizing the norm of this is the same as maximizing the norm squared. Then we are trying to maximize the sum
          begin{equation}
          S = sum_{i=1}^{n} lambda_i^2v_i^2
          end{equation}

          under the constraint that $v$ is a unit vector (i.e., $sum_i v_i^2 = 1$). The maximum is attained by finding the largest $lambda_i^2$ and setting its corresponding $v_i$ to $1$ and then setting each other $v_j$ to $0$. Then the maximum of $S$ (which is the norm squared) is the square of the absolutely largest eigenvalue of $A$. Taking the square root, we get the absolutely largest eigenvalue of $A$.






          share|cite|improve this answer











          $endgroup$













          • $begingroup$
            "Since $Dv$ has only nonzero entries..." I don't understand much in this section. Why is the maximum of $||Dv||$ attained when we select $e_n$, what does "pick off" refer to, and what's SVD?
            $endgroup$
            – Erik Vesterlund
            Dec 11 '13 at 22:38










          • $begingroup$
            I've added some details (and removed a few confusing things) to make this a little clearer.
            $endgroup$
            – yoknapatawpha
            Dec 12 '13 at 0:10










          • $begingroup$
            This is overkill. See @user42070's more elementary approach.
            $endgroup$
            – Ted Shifrin
            Dec 12 '13 at 0:38






          • 1




            $begingroup$
            @TedShifrin is it not up to me who asked the question to determine what is overkill? I asked for more details, the answer is not overkill, it's precisely what I needed.
            $endgroup$
            – Erik Vesterlund
            Dec 12 '13 at 13:50






          • 1




            $begingroup$
            @ErikVesterlund If you consider $S$ just as a weighted sum (putting aside for now that these are eigenvalues of $A$ we're dealing with), then intuitively it makes sense that to maximize the weighted sum, we'd assign all of the weight to the largest element in the sum. Considering the case of a $2 times 2$ matrix, we'd just have $lambda_1$ and $lambda_2$ so that $S = lambda_1^2v_1^2 + lambda_2^2v_2^2$. Here, if $lambda_1 > lambda_2$, we'd maximize $S$ by setting $v_1 = 1$ and $v_2 = 0$ (and vice-versa if $lambda_2 > lambda_1$). The case of higher dimensions is similar. Does this help?
            $endgroup$
            – yoknapatawpha
            Dec 12 '13 at 20:33


















          21












          $begingroup$

          The norm of a matrix is defined as
          begin{equation}
          |A| = sup_{|u| = 1} |Au|
          end{equation}

          Taking the singular value decomposition of the matrix $A$, we have
          begin{equation}
          A = VD W^T
          end{equation}

          where $V$ and $W$ are orthonormal and $D$ is a diagonal matrix. Since $V$ and $W$ are orthonormal, we have $|V| = 1$ and $|W| = 1$. Then $|Av| = |D v|$ for any vector $v$. Then we can maximize the norm of $Av$ by maximizing the norm of $Dv$.



          By the definition of singular value decomposition, $D$ will have the singular values of $A$ on its main diagonal and will have zeros everywhere else. Let $lambda_1, ldots, lambda_n$ denote these diagonal entries so that



          begin{equation}
          D = left(begin{array}{cccc}
          lambda_1 & 0 & ldots & 0 \
          0 & lambda_2 & ldots & 0 \
          vdots & & ddots & vdots \
          0 & 0 & ldots & lambda_n
          end{array}right)
          end{equation}



          Taking some $v = (v_1, v_2, ldots, v_n)^T$, the product $Dv$ takes the form
          begin{equation}
          Dv = left(begin{array}{c}
          lambda_1v_1 \
          vdots \
          lambda_nv_n
          end{array}right)
          end{equation}

          Maximizing the norm of this is the same as maximizing the norm squared. Then we are trying to maximize the sum
          begin{equation}
          S = sum_{i=1}^{n} lambda_i^2v_i^2
          end{equation}

          under the constraint that $v$ is a unit vector (i.e., $sum_i v_i^2 = 1$). The maximum is attained by finding the largest $lambda_i^2$ and setting its corresponding $v_i$ to $1$ and then setting each other $v_j$ to $0$. Then the maximum of $S$ (which is the norm squared) is the square of the absolutely largest eigenvalue of $A$. Taking the square root, we get the absolutely largest eigenvalue of $A$.






          share|cite|improve this answer











          $endgroup$













          • $begingroup$
            "Since $Dv$ has only nonzero entries..." I don't understand much in this section. Why is the maximum of $||Dv||$ attained when we select $e_n$, what does "pick off" refer to, and what's SVD?
            $endgroup$
            – Erik Vesterlund
            Dec 11 '13 at 22:38










          • $begingroup$
            I've added some details (and removed a few confusing things) to make this a little clearer.
            $endgroup$
            – yoknapatawpha
            Dec 12 '13 at 0:10










          • $begingroup$
            This is overkill. See @user42070's more elementary approach.
            $endgroup$
            – Ted Shifrin
            Dec 12 '13 at 0:38






          • 1




            $begingroup$
            @TedShifrin is it not up to me who asked the question to determine what is overkill? I asked for more details, the answer is not overkill, it's precisely what I needed.
            $endgroup$
            – Erik Vesterlund
            Dec 12 '13 at 13:50






          • 1




            $begingroup$
            @ErikVesterlund If you consider $S$ just as a weighted sum (putting aside for now that these are eigenvalues of $A$ we're dealing with), then intuitively it makes sense that to maximize the weighted sum, we'd assign all of the weight to the largest element in the sum. Considering the case of a $2 times 2$ matrix, we'd just have $lambda_1$ and $lambda_2$ so that $S = lambda_1^2v_1^2 + lambda_2^2v_2^2$. Here, if $lambda_1 > lambda_2$, we'd maximize $S$ by setting $v_1 = 1$ and $v_2 = 0$ (and vice-versa if $lambda_2 > lambda_1$). The case of higher dimensions is similar. Does this help?
            $endgroup$
            – yoknapatawpha
            Dec 12 '13 at 20:33
















          21












          21








          21





          $begingroup$

          The norm of a matrix is defined as
          begin{equation}
          |A| = sup_{|u| = 1} |Au|
          end{equation}

          Taking the singular value decomposition of the matrix $A$, we have
          begin{equation}
          A = VD W^T
          end{equation}

          where $V$ and $W$ are orthonormal and $D$ is a diagonal matrix. Since $V$ and $W$ are orthonormal, we have $|V| = 1$ and $|W| = 1$. Then $|Av| = |D v|$ for any vector $v$. Then we can maximize the norm of $Av$ by maximizing the norm of $Dv$.



          By the definition of singular value decomposition, $D$ will have the singular values of $A$ on its main diagonal and will have zeros everywhere else. Let $lambda_1, ldots, lambda_n$ denote these diagonal entries so that



          begin{equation}
          D = left(begin{array}{cccc}
          lambda_1 & 0 & ldots & 0 \
          0 & lambda_2 & ldots & 0 \
          vdots & & ddots & vdots \
          0 & 0 & ldots & lambda_n
          end{array}right)
          end{equation}



          Taking some $v = (v_1, v_2, ldots, v_n)^T$, the product $Dv$ takes the form
          begin{equation}
          Dv = left(begin{array}{c}
          lambda_1v_1 \
          vdots \
          lambda_nv_n
          end{array}right)
          end{equation}

          Maximizing the norm of this is the same as maximizing the norm squared. Then we are trying to maximize the sum
          begin{equation}
          S = sum_{i=1}^{n} lambda_i^2v_i^2
          end{equation}

          under the constraint that $v$ is a unit vector (i.e., $sum_i v_i^2 = 1$). The maximum is attained by finding the largest $lambda_i^2$ and setting its corresponding $v_i$ to $1$ and then setting each other $v_j$ to $0$. Then the maximum of $S$ (which is the norm squared) is the square of the absolutely largest eigenvalue of $A$. Taking the square root, we get the absolutely largest eigenvalue of $A$.






          share|cite|improve this answer











          $endgroup$



          The norm of a matrix is defined as
          begin{equation}
          |A| = sup_{|u| = 1} |Au|
          end{equation}

          Taking the singular value decomposition of the matrix $A$, we have
          begin{equation}
          A = VD W^T
          end{equation}

          where $V$ and $W$ are orthonormal and $D$ is a diagonal matrix. Since $V$ and $W$ are orthonormal, we have $|V| = 1$ and $|W| = 1$. Then $|Av| = |D v|$ for any vector $v$. Then we can maximize the norm of $Av$ by maximizing the norm of $Dv$.



          By the definition of singular value decomposition, $D$ will have the singular values of $A$ on its main diagonal and will have zeros everywhere else. Let $lambda_1, ldots, lambda_n$ denote these diagonal entries so that



          begin{equation}
          D = left(begin{array}{cccc}
          lambda_1 & 0 & ldots & 0 \
          0 & lambda_2 & ldots & 0 \
          vdots & & ddots & vdots \
          0 & 0 & ldots & lambda_n
          end{array}right)
          end{equation}



          Taking some $v = (v_1, v_2, ldots, v_n)^T$, the product $Dv$ takes the form
          begin{equation}
          Dv = left(begin{array}{c}
          lambda_1v_1 \
          vdots \
          lambda_nv_n
          end{array}right)
          end{equation}

          Maximizing the norm of this is the same as maximizing the norm squared. Then we are trying to maximize the sum
          begin{equation}
          S = sum_{i=1}^{n} lambda_i^2v_i^2
          end{equation}

          under the constraint that $v$ is a unit vector (i.e., $sum_i v_i^2 = 1$). The maximum is attained by finding the largest $lambda_i^2$ and setting its corresponding $v_i$ to $1$ and then setting each other $v_j$ to $0$. Then the maximum of $S$ (which is the norm squared) is the square of the absolutely largest eigenvalue of $A$. Taking the square root, we get the absolutely largest eigenvalue of $A$.







          share|cite|improve this answer














          share|cite|improve this answer



          share|cite|improve this answer








          edited Jan 4 at 23:33









          Wangkun Xu

          485




          485










          answered Dec 11 '13 at 22:09









          yoknapatawphayoknapatawpha

          3,23172342




          3,23172342












          • $begingroup$
            "Since $Dv$ has only nonzero entries..." I don't understand much in this section. Why is the maximum of $||Dv||$ attained when we select $e_n$, what does "pick off" refer to, and what's SVD?
            $endgroup$
            – Erik Vesterlund
            Dec 11 '13 at 22:38










          • $begingroup$
            I've added some details (and removed a few confusing things) to make this a little clearer.
            $endgroup$
            – yoknapatawpha
            Dec 12 '13 at 0:10










          • $begingroup$
            This is overkill. See @user42070's more elementary approach.
            $endgroup$
            – Ted Shifrin
            Dec 12 '13 at 0:38






          • 1




            $begingroup$
            @TedShifrin is it not up to me who asked the question to determine what is overkill? I asked for more details, the answer is not overkill, it's precisely what I needed.
            $endgroup$
            – Erik Vesterlund
            Dec 12 '13 at 13:50






          • 1




            $begingroup$
            @ErikVesterlund If you consider $S$ just as a weighted sum (putting aside for now that these are eigenvalues of $A$ we're dealing with), then intuitively it makes sense that to maximize the weighted sum, we'd assign all of the weight to the largest element in the sum. Considering the case of a $2 times 2$ matrix, we'd just have $lambda_1$ and $lambda_2$ so that $S = lambda_1^2v_1^2 + lambda_2^2v_2^2$. Here, if $lambda_1 > lambda_2$, we'd maximize $S$ by setting $v_1 = 1$ and $v_2 = 0$ (and vice-versa if $lambda_2 > lambda_1$). The case of higher dimensions is similar. Does this help?
            $endgroup$
            – yoknapatawpha
            Dec 12 '13 at 20:33




















          • $begingroup$
            "Since $Dv$ has only nonzero entries..." I don't understand much in this section. Why is the maximum of $||Dv||$ attained when we select $e_n$, what does "pick off" refer to, and what's SVD?
            $endgroup$
            – Erik Vesterlund
            Dec 11 '13 at 22:38










          • $begingroup$
            I've added some details (and removed a few confusing things) to make this a little clearer.
            $endgroup$
            – yoknapatawpha
            Dec 12 '13 at 0:10










          • $begingroup$
            This is overkill. See @user42070's more elementary approach.
            $endgroup$
            – Ted Shifrin
            Dec 12 '13 at 0:38






          • 1




            $begingroup$
            @TedShifrin is it not up to me who asked the question to determine what is overkill? I asked for more details, the answer is not overkill, it's precisely what I needed.
            $endgroup$
            – Erik Vesterlund
            Dec 12 '13 at 13:50






          • 1




            $begingroup$
            @ErikVesterlund If you consider $S$ just as a weighted sum (putting aside for now that these are eigenvalues of $A$ we're dealing with), then intuitively it makes sense that to maximize the weighted sum, we'd assign all of the weight to the largest element in the sum. Considering the case of a $2 times 2$ matrix, we'd just have $lambda_1$ and $lambda_2$ so that $S = lambda_1^2v_1^2 + lambda_2^2v_2^2$. Here, if $lambda_1 > lambda_2$, we'd maximize $S$ by setting $v_1 = 1$ and $v_2 = 0$ (and vice-versa if $lambda_2 > lambda_1$). The case of higher dimensions is similar. Does this help?
            $endgroup$
            – yoknapatawpha
            Dec 12 '13 at 20:33


















          $begingroup$
          "Since $Dv$ has only nonzero entries..." I don't understand much in this section. Why is the maximum of $||Dv||$ attained when we select $e_n$, what does "pick off" refer to, and what's SVD?
          $endgroup$
          – Erik Vesterlund
          Dec 11 '13 at 22:38




          $begingroup$
          "Since $Dv$ has only nonzero entries..." I don't understand much in this section. Why is the maximum of $||Dv||$ attained when we select $e_n$, what does "pick off" refer to, and what's SVD?
          $endgroup$
          – Erik Vesterlund
          Dec 11 '13 at 22:38












          $begingroup$
          I've added some details (and removed a few confusing things) to make this a little clearer.
          $endgroup$
          – yoknapatawpha
          Dec 12 '13 at 0:10




          $begingroup$
          I've added some details (and removed a few confusing things) to make this a little clearer.
          $endgroup$
          – yoknapatawpha
          Dec 12 '13 at 0:10












          $begingroup$
          This is overkill. See @user42070's more elementary approach.
          $endgroup$
          – Ted Shifrin
          Dec 12 '13 at 0:38




          $begingroup$
          This is overkill. See @user42070's more elementary approach.
          $endgroup$
          – Ted Shifrin
          Dec 12 '13 at 0:38




          1




          1




          $begingroup$
          @TedShifrin is it not up to me who asked the question to determine what is overkill? I asked for more details, the answer is not overkill, it's precisely what I needed.
          $endgroup$
          – Erik Vesterlund
          Dec 12 '13 at 13:50




          $begingroup$
          @TedShifrin is it not up to me who asked the question to determine what is overkill? I asked for more details, the answer is not overkill, it's precisely what I needed.
          $endgroup$
          – Erik Vesterlund
          Dec 12 '13 at 13:50




          1




          1




          $begingroup$
          @ErikVesterlund If you consider $S$ just as a weighted sum (putting aside for now that these are eigenvalues of $A$ we're dealing with), then intuitively it makes sense that to maximize the weighted sum, we'd assign all of the weight to the largest element in the sum. Considering the case of a $2 times 2$ matrix, we'd just have $lambda_1$ and $lambda_2$ so that $S = lambda_1^2v_1^2 + lambda_2^2v_2^2$. Here, if $lambda_1 > lambda_2$, we'd maximize $S$ by setting $v_1 = 1$ and $v_2 = 0$ (and vice-versa if $lambda_2 > lambda_1$). The case of higher dimensions is similar. Does this help?
          $endgroup$
          – yoknapatawpha
          Dec 12 '13 at 20:33






          $begingroup$
          @ErikVesterlund If you consider $S$ just as a weighted sum (putting aside for now that these are eigenvalues of $A$ we're dealing with), then intuitively it makes sense that to maximize the weighted sum, we'd assign all of the weight to the largest element in the sum. Considering the case of a $2 times 2$ matrix, we'd just have $lambda_1$ and $lambda_2$ so that $S = lambda_1^2v_1^2 + lambda_2^2v_2^2$. Here, if $lambda_1 > lambda_2$, we'd maximize $S$ by setting $v_1 = 1$ and $v_2 = 0$ (and vice-versa if $lambda_2 > lambda_1$). The case of higher dimensions is similar. Does this help?
          $endgroup$
          – yoknapatawpha
          Dec 12 '13 at 20:33













          15












          $begingroup$

          But the key point is exactly that your matrix is diagonalizable more specially that you can find an orthonormal basis of eigenvector $e_i$ for which you have $A e_i=lambda_i e_i$.



          Then your write for $x=sum x_ie_i$ and you have $Ax=sum x_ilambda_i e_i$ so that $|Ax|^2=sumlambda_i^2x_i^2$ now by definition of the norm of matrix it gives you $$frac{|Ax|}{|x|}leq|lambda_{i_0}|$$ therefore $|A|leq|lambda_{i_0}|$ where $lambda_{i_0}$ is the greatest eigenvalue . Finally the identity $|Ae_{i_0}|=|lambda_{i_0}e_i|$ gives you the inequality $|A|geq|lambda_{i_0}|$.






          share|cite|improve this answer











          $endgroup$













          • $begingroup$
            How do you get that $||Ax||^2 = sum lambda_i^2 x_i^2$? At least in my book the matrix norm is not defined, not previous to the exercise anyway so, what is it?
            $endgroup$
            – Erik Vesterlund
            Dec 11 '13 at 22:36






          • 1




            $begingroup$
            just because $(e_i)$ is an orthonormal basis, the whole question relies on the euclidean structure : the norm in $mathbb{R}^n$ is the one defined by $|x|^2:=<x,x>=sum x_i^2$. The fact that $e_i$ is orthonormal reads $<e_i,e_j>=delta_{ij}$ which means $1$ if $i=j$ and $0$ otherwise. Now compute $|x|^2=<x,x>=sum x_ix_j<e_i,e_j>=sum x_i^2$
            $endgroup$
            – user42070
            Dec 11 '13 at 22:44












          • $begingroup$
            Is it correct to draw the conclusion, that when we can find an eigenbasis, then the largest singular value always equals the largest eigenvalue?
            $endgroup$
            – Thomas Ahle
            Feb 14 '17 at 6:24










          • $begingroup$
            So $x_i$ is not the elements of $x$ but its decomposition in the orthonormal basis $(e_i)$?
            $endgroup$
            – Ella Shar
            Jan 27 at 8:55


















          15












          $begingroup$

          But the key point is exactly that your matrix is diagonalizable more specially that you can find an orthonormal basis of eigenvector $e_i$ for which you have $A e_i=lambda_i e_i$.



          Then your write for $x=sum x_ie_i$ and you have $Ax=sum x_ilambda_i e_i$ so that $|Ax|^2=sumlambda_i^2x_i^2$ now by definition of the norm of matrix it gives you $$frac{|Ax|}{|x|}leq|lambda_{i_0}|$$ therefore $|A|leq|lambda_{i_0}|$ where $lambda_{i_0}$ is the greatest eigenvalue . Finally the identity $|Ae_{i_0}|=|lambda_{i_0}e_i|$ gives you the inequality $|A|geq|lambda_{i_0}|$.






          share|cite|improve this answer











          $endgroup$













          • $begingroup$
            How do you get that $||Ax||^2 = sum lambda_i^2 x_i^2$? At least in my book the matrix norm is not defined, not previous to the exercise anyway so, what is it?
            $endgroup$
            – Erik Vesterlund
            Dec 11 '13 at 22:36






          • 1




            $begingroup$
            just because $(e_i)$ is an orthonormal basis, the whole question relies on the euclidean structure : the norm in $mathbb{R}^n$ is the one defined by $|x|^2:=<x,x>=sum x_i^2$. The fact that $e_i$ is orthonormal reads $<e_i,e_j>=delta_{ij}$ which means $1$ if $i=j$ and $0$ otherwise. Now compute $|x|^2=<x,x>=sum x_ix_j<e_i,e_j>=sum x_i^2$
            $endgroup$
            – user42070
            Dec 11 '13 at 22:44












          • $begingroup$
            Is it correct to draw the conclusion, that when we can find an eigenbasis, then the largest singular value always equals the largest eigenvalue?
            $endgroup$
            – Thomas Ahle
            Feb 14 '17 at 6:24










          • $begingroup$
            So $x_i$ is not the elements of $x$ but its decomposition in the orthonormal basis $(e_i)$?
            $endgroup$
            – Ella Shar
            Jan 27 at 8:55
















          15












          15








          15





          $begingroup$

          But the key point is exactly that your matrix is diagonalizable more specially that you can find an orthonormal basis of eigenvector $e_i$ for which you have $A e_i=lambda_i e_i$.



          Then your write for $x=sum x_ie_i$ and you have $Ax=sum x_ilambda_i e_i$ so that $|Ax|^2=sumlambda_i^2x_i^2$ now by definition of the norm of matrix it gives you $$frac{|Ax|}{|x|}leq|lambda_{i_0}|$$ therefore $|A|leq|lambda_{i_0}|$ where $lambda_{i_0}$ is the greatest eigenvalue . Finally the identity $|Ae_{i_0}|=|lambda_{i_0}e_i|$ gives you the inequality $|A|geq|lambda_{i_0}|$.






          share|cite|improve this answer











          $endgroup$



          But the key point is exactly that your matrix is diagonalizable more specially that you can find an orthonormal basis of eigenvector $e_i$ for which you have $A e_i=lambda_i e_i$.



          Then your write for $x=sum x_ie_i$ and you have $Ax=sum x_ilambda_i e_i$ so that $|Ax|^2=sumlambda_i^2x_i^2$ now by definition of the norm of matrix it gives you $$frac{|Ax|}{|x|}leq|lambda_{i_0}|$$ therefore $|A|leq|lambda_{i_0}|$ where $lambda_{i_0}$ is the greatest eigenvalue . Finally the identity $|Ae_{i_0}|=|lambda_{i_0}e_i|$ gives you the inequality $|A|geq|lambda_{i_0}|$.







          share|cite|improve this answer














          share|cite|improve this answer



          share|cite|improve this answer








          edited Dec 11 '13 at 22:48

























          answered Dec 11 '13 at 22:10









          user42070user42070

          40028




          40028












          • $begingroup$
            How do you get that $||Ax||^2 = sum lambda_i^2 x_i^2$? At least in my book the matrix norm is not defined, not previous to the exercise anyway so, what is it?
            $endgroup$
            – Erik Vesterlund
            Dec 11 '13 at 22:36






          • 1




            $begingroup$
            just because $(e_i)$ is an orthonormal basis, the whole question relies on the euclidean structure : the norm in $mathbb{R}^n$ is the one defined by $|x|^2:=<x,x>=sum x_i^2$. The fact that $e_i$ is orthonormal reads $<e_i,e_j>=delta_{ij}$ which means $1$ if $i=j$ and $0$ otherwise. Now compute $|x|^2=<x,x>=sum x_ix_j<e_i,e_j>=sum x_i^2$
            $endgroup$
            – user42070
            Dec 11 '13 at 22:44












          • $begingroup$
            Is it correct to draw the conclusion, that when we can find an eigenbasis, then the largest singular value always equals the largest eigenvalue?
            $endgroup$
            – Thomas Ahle
            Feb 14 '17 at 6:24










          • $begingroup$
            So $x_i$ is not the elements of $x$ but its decomposition in the orthonormal basis $(e_i)$?
            $endgroup$
            – Ella Shar
            Jan 27 at 8:55




















          • $begingroup$
            How do you get that $||Ax||^2 = sum lambda_i^2 x_i^2$? At least in my book the matrix norm is not defined, not previous to the exercise anyway so, what is it?
            $endgroup$
            – Erik Vesterlund
            Dec 11 '13 at 22:36






          • 1




            $begingroup$
            just because $(e_i)$ is an orthonormal basis, the whole question relies on the euclidean structure : the norm in $mathbb{R}^n$ is the one defined by $|x|^2:=<x,x>=sum x_i^2$. The fact that $e_i$ is orthonormal reads $<e_i,e_j>=delta_{ij}$ which means $1$ if $i=j$ and $0$ otherwise. Now compute $|x|^2=<x,x>=sum x_ix_j<e_i,e_j>=sum x_i^2$
            $endgroup$
            – user42070
            Dec 11 '13 at 22:44












          • $begingroup$
            Is it correct to draw the conclusion, that when we can find an eigenbasis, then the largest singular value always equals the largest eigenvalue?
            $endgroup$
            – Thomas Ahle
            Feb 14 '17 at 6:24










          • $begingroup$
            So $x_i$ is not the elements of $x$ but its decomposition in the orthonormal basis $(e_i)$?
            $endgroup$
            – Ella Shar
            Jan 27 at 8:55


















          $begingroup$
          How do you get that $||Ax||^2 = sum lambda_i^2 x_i^2$? At least in my book the matrix norm is not defined, not previous to the exercise anyway so, what is it?
          $endgroup$
          – Erik Vesterlund
          Dec 11 '13 at 22:36




          $begingroup$
          How do you get that $||Ax||^2 = sum lambda_i^2 x_i^2$? At least in my book the matrix norm is not defined, not previous to the exercise anyway so, what is it?
          $endgroup$
          – Erik Vesterlund
          Dec 11 '13 at 22:36




          1




          1




          $begingroup$
          just because $(e_i)$ is an orthonormal basis, the whole question relies on the euclidean structure : the norm in $mathbb{R}^n$ is the one defined by $|x|^2:=<x,x>=sum x_i^2$. The fact that $e_i$ is orthonormal reads $<e_i,e_j>=delta_{ij}$ which means $1$ if $i=j$ and $0$ otherwise. Now compute $|x|^2=<x,x>=sum x_ix_j<e_i,e_j>=sum x_i^2$
          $endgroup$
          – user42070
          Dec 11 '13 at 22:44






          $begingroup$
          just because $(e_i)$ is an orthonormal basis, the whole question relies on the euclidean structure : the norm in $mathbb{R}^n$ is the one defined by $|x|^2:=<x,x>=sum x_i^2$. The fact that $e_i$ is orthonormal reads $<e_i,e_j>=delta_{ij}$ which means $1$ if $i=j$ and $0$ otherwise. Now compute $|x|^2=<x,x>=sum x_ix_j<e_i,e_j>=sum x_i^2$
          $endgroup$
          – user42070
          Dec 11 '13 at 22:44














          $begingroup$
          Is it correct to draw the conclusion, that when we can find an eigenbasis, then the largest singular value always equals the largest eigenvalue?
          $endgroup$
          – Thomas Ahle
          Feb 14 '17 at 6:24




          $begingroup$
          Is it correct to draw the conclusion, that when we can find an eigenbasis, then the largest singular value always equals the largest eigenvalue?
          $endgroup$
          – Thomas Ahle
          Feb 14 '17 at 6:24












          $begingroup$
          So $x_i$ is not the elements of $x$ but its decomposition in the orthonormal basis $(e_i)$?
          $endgroup$
          – Ella Shar
          Jan 27 at 8:55






          $begingroup$
          So $x_i$ is not the elements of $x$ but its decomposition in the orthonormal basis $(e_i)$?
          $endgroup$
          – Ella Shar
          Jan 27 at 8:55




















          draft saved

          draft discarded




















































          Thanks for contributing an answer to Mathematics Stack Exchange!


          • Please be sure to answer the question. Provide details and share your research!

          But avoid



          • Asking for help, clarification, or responding to other answers.

          • Making statements based on opinion; back them up with references or personal experience.


          Use MathJax to format equations. MathJax reference.


          To learn more, see our tips on writing great answers.




          draft saved


          draft discarded














          StackExchange.ready(
          function () {
          StackExchange.openid.initPostLogin('.new-post-login', 'https%3a%2f%2fmath.stackexchange.com%2fquestions%2f603375%2fnorm-of-a-symmetric-matrix-equals-spectral-radius%23new-answer', 'question_page');
          }
          );

          Post as a guest















          Required, but never shown





















































          Required, but never shown














          Required, but never shown












          Required, but never shown







          Required, but never shown

































          Required, but never shown














          Required, but never shown












          Required, but never shown







          Required, but never shown







          Popular posts from this blog

          How do I know what Microsoft account the skydrive app is syncing to?

          When does type information flow backwards in C++?

          Grease: Live!